Continuité sur un segment

Exercices du dossier Continuité sur un segment

Exercice 244 *

13 janvier 2021 20:53 — Par Alain Soyeur François Capaces Emmanuel Vieillard-Baron

Soient \(f,g:[a,b] \rightarrow \mathbb{R}\) continues telles que \(\forall x \in [a,b], \quad f(x) < g(x)\). Montrer qu’il existe \(\alpha >0\) tel que \[\forall x \in [a,b], \quad f(x) \leqslant g(x)-\alpha.\]



[ID: 659] [Date de publication: 13 janvier 2021 20:53] [Catégorie(s): Continuité sur un segment ] [ Nombre commentaires: 0] [nombre d'éditeurs: 1 ] [Editeur(s): Emmanuel Vieillard-Baron ] [nombre d'auteurs: 3 ] [Auteur(s): Alain Soyeur François Capaces Emmanuel Vieillard-Baron ]
Accordéon
Titre
Solution
Texte

Exercice 244
Par Alain Soyeur François Capaces Emmanuel Vieillard-Baron le 13 janvier 2021 20:54

Considérons la fonction définie sur \(\left[a,b\right]\) par \(\theta=g-f\).La fonction \(\theta\) est continue et strictement positive sur \([a,b]\). Elle possède donc un minimum strictement positif atteint en un certain point \(c\in[a,b]\). Posons : \(\alpha=\theta(c)=g(c)-f(c)>0\). Il est clair que : \(\forall x \in [a,b], \quad f(x) \leqslant g(x)-\alpha.\)


Exercice 543 *

13 janvier 2021 20:54 — Par Alain Soyeur François Capaces Emmanuel Vieillard-Baron

Soient deux fonctions \(f, g : [0, 1] \mapsto \mathbb{R}\) continues telles que \[\forall x \in [0, 1],~ 0 < f(x) < g(x)\] Montrez qu’il existe un réel \(k > 1\) tel que \[\forall x \in [0, 1],~g(x) \geqslant k f(x)\]



[ID: 661] [Date de publication: 13 janvier 2021 20:54] [Catégorie(s): Continuité sur un segment ] [ Nombre commentaires: 0] [nombre d'éditeurs: 1 ] [Editeur(s): Emmanuel Vieillard-Baron ] [nombre d'auteurs: 3 ] [Auteur(s): Alain Soyeur François Capaces Emmanuel Vieillard-Baron ]
Accordéon
Titre
Solution
Texte

Exercice 543
Par Alain Soyeur François Capaces Emmanuel Vieillard-Baron le 13 janvier 2021 20:54

Considérons la fonction \(\varphi\) définie sur le segment \([0, 1]\) par \(\varphi(x) = g(x) / f(x)\). Comme la fonction \(f\) ne s’annule pas, \(\varphi\) est définie et continue sur le segment \([0, 1]\) et possède donc un minimum. Il existe \(x_0 \in [0, 1]\) tel que \(\forall x \in [0, 1]\), \(\varphi(x) \geqslant\varphi(x_0)\). Posons \(k = \varphi(x_0)\). Comme \(f(x_0) < g(x_0)\), \(k > 1\) et alors \(\forall x \in [0, 1]\), \(g(x) / f(x) \geqslant k\) d’où le résultat.


Exercice 329 *

13 janvier 2021 20:54 — Par Alain Soyeur François Capaces Emmanuel Vieillard-Baron

Soit une fonction \(f\) continue sur \(\mathbb{R}\). On suppose que \[f(x) \xrightarrow[x \rightarrow +\infty]{} +\infty \textrm{ et } f(x) \xrightarrow[x \rightarrow -\infty]{} +\infty\] Montrez que la fonction \(f\) possède un minimum.



[ID: 663] [Date de publication: 13 janvier 2021 20:54] [Catégorie(s): Continuité sur un segment ] [ Nombre commentaires: 0] [nombre d'éditeurs: 1 ] [Editeur(s): Emmanuel Vieillard-Baron ] [nombre d'auteurs: 3 ] [Auteur(s): Alain Soyeur François Capaces Emmanuel Vieillard-Baron ]
Accordéon
Titre
Solution
Texte

Exercice 329
Par Alain Soyeur François Capaces Emmanuel Vieillard-Baron le 13 janvier 2021 20:54

Soit \(x_0 \in \mathbb{R}\). Posons \(A = f(x_0)\). Comme \(f(x) \xrightarrow[x \rightarrow \pm\infty]{} +\infty\), d’après la définition de la limite, il existe \(B > 0\) tel que \(\forall x \in \mathbb{R}\), \(\lvert x \rvert \geqslant B \Rightarrow f(x) \geqslant A\). On a en particulier \(x_0 \in [-B, B]\). La fonction \(f\) est continue sur le segment \([-B, B]\) et donc possède un minimum sur ce segment : \[\exists c \in [-B, B] \mid \forall x \in [-B, B],~ f(c) \leqslant f(x)\] Montrons que \(\forall x \in \mathbb{R}\), \(f(x) \geqslant f(c)\) ce qui montrera que \(f(c)\) est un minimum de \(f\) sur \(\mathbb{R}\). Soit \(x \in \mathbb{R}\). Si \(x \in [-B, B]\), on a bien \(f(c) \leqslant f(x)\). Si \(x \not\in [-B, B]\), alors \(f(x) \geqslant A = f(x_0)\) et comme \(x_0 \in [-B, B]\), il vient que \(f(x) \geqslant f(x_0) \geqslant f(c)\).


Exercice 297 *

13 janvier 2021 20:54 — Par Alain Soyeur François Capaces Emmanuel Vieillard-Baron

Soit \(f:\mathbb{R} \mapsto \mathbb{R}\) une application T-périodique (\(T>0\)) et continue. Montrer que \(f\) est bornée.



[ID: 665] [Date de publication: 13 janvier 2021 20:54] [Catégorie(s): Continuité sur un segment ] [ Nombre commentaires: 0] [nombre d'éditeurs: 1 ] [Editeur(s): Emmanuel Vieillard-Baron ] [nombre d'auteurs: 3 ] [Auteur(s): Alain Soyeur François Capaces Emmanuel Vieillard-Baron ]
Accordéon
Titre
Solution
Texte

Exercice 297
Par Alain Soyeur François Capaces Emmanuel Vieillard-Baron le 13 janvier 2021 20:54

Considérons la restriction de \(f\) au segment \([0,T]\). Elle est continue sur ce segment, et donc est bornée: \(\exists M>0~:\quad \forall x\in [0,T],\quad \left| f(x) \right| \leqslant M\). Mais alors, si \(x\in \mathbb{R}\), avec \(y=E(\dfrac{x}{T})\), on a \(nT\leqslant x < (n+1)T\) et donc \(f(x)=f(x-nT)\) et puisque \(x-nT \in [0,T]\), \(\left| f(x) \right| \leqslant M\).


Exercice 381 *

13 janvier 2021 20:54 — Par Alain Soyeur François Capaces Emmanuel Vieillard-Baron

Soit \(f: \mathbb{R}\rightarrow \mathbb{R}\) bornée et \(g: \mathbb{R}\rightarrow \mathbb{R}\) continue. Montrer que \(g \circ f\) et \(f \circ g\) sont bornées.



[ID: 667] [Date de publication: 13 janvier 2021 20:54] [Catégorie(s): Continuité sur un segment ] [ Nombre commentaires: 0] [nombre d'éditeurs: 1 ] [Editeur(s): Emmanuel Vieillard-Baron ] [nombre d'auteurs: 3 ] [Auteur(s): Alain Soyeur François Capaces Emmanuel Vieillard-Baron ]
Accordéon
Titre
Solution
Texte

Exercice 381
Par Alain Soyeur François Capaces Emmanuel Vieillard-Baron le 13 janvier 2021 20:54

Comme \(f\) est bornée sur \(\mathbb{R}\), il existe \(M >0\) tel que pour tout \(x \in \mathbb{R}\), \(\left|f(x)\right| \leqslant M\). En particulier, \(\forall x\in \mathbb R, \quad f\left(g\left(x\right)\right) \leqslant M\) donc \(f \circ g\) est bornée sur \(\mathbb{R}\). Par ailleurs \(f(\mathbb{R}) \subset [-M,M]\) et \(g\) est continue sur \([-M,M]\) donc \(g\) est majorée et minorée sur \([-M,M]\) et \(g\circ f\) est bornée sur \(\mathbb{R}\).


Accordéon
Titre
Solution
Texte

Exercice 692
Par Alain Soyeur François Capaces Emmanuel Vieillard-Baron le 13 janvier 2021 20:54

La fonction \(\varphi(x)=\dfrac{f(x)}{g(x)}\) est continue sur le segment \([0,1]\). Elle est donc bornée et atteint ses bornes sur ce segment: il existe \(x_0 \in [0,1]\) tel que \(\sup_{x\in [0,1]} \dfrac{ f(x)}{g(x)} = \dfrac{ f(x_0)}{g(x_0)}=k <1\). Donc \[\forall x\in [0,1], \quad 0< \dfrac{f(x)}{g(x)} \leqslant k <1\] Alors \(\forall n \in \mathbb N\), \(0\leqslant a_n \leqslant k^n\) et comme \(k<1\), la suite géométrique \((k^n)\) converge vers \(0\). D’après le théorème des gendarmes, la suite \((a_n)\) converge vers \(0\).


Accordéon
Titre
Solution
Texte

Exercice 440
Par Alain Soyeur François Capaces Emmanuel Vieillard-Baron le 13 janvier 2021 20:54

La fonction \(\varphi\) est continue comme somme et valeur absolue de fonctions continues. Elle est donc continue en tout point de l’ensemble \(K\). Comme la fonction \(\varphi\) est continue sur le segment \([a, b]\), elle possède un minimum \(M_1 = \varphi(x_1)\) sur \([a, b]\) avec \(x_1 \in [a, b]\). De même, elle possède un minimum \(M_2\) sur le segment \([c, d]\) avec \(M_2 = \varphi(x_2)\)\(x_2 \in [c, d]\). On pose \(m = \min\{M_1, M_2\}\) et on vérifie que \(M\) est un minimum de la fonction \(\varphi\) sur \(K\). Donc on a montré que \[\exists x_0 \in K ~:\quad \forall x \in K,~ \varphi(x_0) \leqslant\varphi(x)\] Si par l’absurde, \(\varphi(x_0) \neq 0\), en utilisant l’inégalité de l’énoncé, puisque \(f(x_0) \neq x_0\), et \(f(x_0) \in K\), on aurait \[\Bigl|f\bigl(f(x_0)\bigr) - f(x_0)\Bigr| < \lvert f(x_0) - x_0 \rvert\] et donc \(\varphi\bigl(f(x_0)\bigr) < \varphi(x_0)\) ce qui est impossible. Par conséquent, \(\varphi(x_0) = 0\) et donc \(f(x_0) = x_0\). Montrons l’unicité du point fixe. S’il existait deux points fixes \(x_1\in K\) et \(x_2\in K\), avec \(x_1 \neq x_2\) on aurait \[\lvert f(x_1)-f(x_2) \rvert < \lvert x_1 - x_2 \rvert\] et donc \(\lvert x_1 - x_2 \rvert < \lvert x_1-x_2 \rvert\) une absurdité.
Remarque : l’hypothèse \(f\) continue est superflue, puisque \(f\) est lipschitzienne.


;
Success message!